You are on page 1of 17

1983 AIME Problems

Problem 1

Let ,, and all exceed 1, and let be a positive number such that log = 24,
log = 40, and log = 12. Find log

Solutions 1

The logarithmic notation doesn't tell us much, so we'll first convert everything to the
equivalent exponential expressions.

24 = , 40 = , and ()12 = . If we now convert everything to a power of 120, it


will be easy to isolate and .

120 = 5 , 120 = 3 , and ()120 = 10 .

With some substitution, we get 5 3 120 = 10 and log = .

Solution 2

Applying the change of base formula,

Therefore, .

Hence, .

Problem 2

Let , where . Determine the


minimum value taken by for in the interval .
Solution

It is best to get rid of the absolute value first.

Under the given circumstances, we notice that , , and


.

Adding these together, we find that the sum is equal to , of which the minimum value
is attained when .

Edit: can equal or


(for example, if and , ). Thus, our two "cases" are
(if ) and (if ). However, both of these cases give us as
the minimum value for , which indeed is the answer posted above.

Problem 3

What is the product of the real roots of the equation ?

Solution

If we expand by squaring, we get a quartic polynomial, which isn't very helpful.

Instead, we substitute for and our equation becomes .

Now we can square; solving for , we get or . The second solution is


extraneous since is positive. So, we have as the only solution for .
Substituting back in for ,

By Vieta's formulas, the product of the roots is .


Problem 4

A machine shop cutting tool is in the shape of a notched circle, as shown. The radius of the
circle is cm, the length of is 6 cm, and that of is 2 cm. The angle is a
right angle. Find the square of the distance (in centimeters) from to the center of the circle.

Solution 1

Because we are given a right angle, we look for ways to apply the Pythagorean Theorem. Let
the foot of the perpendicular from to be and let the foot of the perpendicular
from to the line be . Let and . We're trying to find .

Applying the Pythagorean Theorem, and .

Thus, , and . We solve this system to get


and , resulting in .
Solution 2

Drop perpendiculars from to ( ), to ( ), and to ( ). Also,


draw the midpoint of .

Then the problem is trivialized. Why?

First notice that by computation, is a isosceles triangle; thus


. Then, notice that . Thus the two blue
triangles are congruent.

So, . As , we subtract and get .


Then the Pythagorean Theorem shows .

Problem 5

Suppose that the sum of the squares of two complex numbers and is and the sum of
the cubes is . What is the largest real value that can have?

Solution
Solution 1

One way to solve this problem seems to be by substitution.


and

Because we are only left with and , substitution won't be too bad. Let
and .

We get and

Because we want the largest possible , let's find an expression for in terms of .

Substituting, . Factored, (the Rational


Root Theorem may be used here, along with synthetic division)

The largest possible solution is therefore .

Solution 2

An alternate way to solve this is to let and .

Because we are looking for a value of that is real, we know that , and thus
.

Expanding will give two equations, since the real and imaginary parts
must match up.

Looking at the imaginary part of that equation, , so , and and are


actually complex conjugates.

Looking at the real part of the equation and plugging in , , or


.

Now, evaluating the real part of , which equals (ignoring the odd
powers of , since they would not result in something in the form of ):

Since we know that , it can be plugged in for in the above equation to


yield:
Since the problem is looking for to be a positive integer, only positive half-
integers (and whole-integers) need to be tested. From the Rational Roots theorem, =
5
10, = 5, = 2 all fail, but does work. Thus, the real part of both numbers is , and
their sum is

Solution 3

Start by assuming x and y were roots of some polynomial of the form So then
2 7
and Substituting = 2 we arrive at the polynomial
From rational root theorem we find the roots to be Since
is the sum of the roots and is maximized when b is -4, the answer is

Problem 6

Let equal . Determine the remainder upon dividing by .

Solution
Solution 1

First, we try to find a relationship between the numbers we're provided with and . We
realize that and both and are greater or less than by .

Expressing the numbers in terms of , we get .

Applying the Binomial Theorem, half of our terms cancel out and we are left with
. We realize that all of these terms are divisible by
except the final term.

After some quick division, our answer is .


Solution 2

Since (the Euler's totient function), by Euler's Totient Theorem,


where . Thus

Alternatively, we could have noted that . This way, we have


, and can finish the same way.

Problem 7

Twenty five of King Arthur's knights are seated at their customary round table. Three of them
are chosen - all choices being equally likely - and are sent of to slay a troublesome dragon.
Let be the probability that at least two of the three had been sitting next to each other. If
is written as a fraction in lowest terms, what is the sum of the numerator and the
denominator?

Solution
Solution 1

We can use Complementary counting by finding the probability that none are sitting next to
each other and subtracting it from .

Imagine the other (indistinguishable) people are already seated, and fixed into place.

We will place , , and with and without the restriction.

There are places to place , followed by places to place , and places to place
after and . Hence, there are ways to place in between these people
with restrictions.

Without restrictions, there are places to place , followed by places to place , and
places to place after and . Hence, there are ways to place in
between these people without restrictions.

Thus, the desired amount is , and the answer is


.

Solution 2

There are configurations for the knights about the table.


There are ways to pick a pair of knights from the trio, and there are ways to
determine which order they are seated. Since these two knights must be attached, we let them
be a single entity, so there are configurations for the entities.

However, this overcounts the instances in which the trio sits together; when all three knights
sit together, then two of the pairs from the previous case are counted. However, we only want
to count this as one case, so we need to subtract the number of instances in which the trio sits
together (as a single entity). There are ways to determine their order, and there are
configurations.

Thus, the answer is , and the answer is .

Solution 3

Number the knights around the table 1-25. There are two possibilities: All three sit next to
each other, or two sit next to each other and one is not sitting next to the other two.

Case 1: All three sit next to each other. In this case, you are picking , ,
... . This makes combinations.

Case 2: Like above, there are ways to pick the pair of knights sitting next to each other.
Once a pair is picked, you cannot pick either of the two adjacent knights. (i.e. if you pick
, you may not pick 4 or 7). Thus, there are ways to pick the third knight,
for a total of combinations.

Thus, you have a total of allowable ways to pick the knights. The

total number of ways to pick the knights is .

The probability is , and the answer is .

Solution 4

Pick an arbitrary spot for the first knight. Then pick spots for the next two knights in order.

Case 1: The second knight sits next to the first knight. There are 2 possible places for this out

of 24, so the probability of this is . We do not need to consider the third knight.

Case 2: The second knight sits two spaces from the first knight. There are 2 possible places

for this out of 24, so the probability is . Then there are 3 places out of a remaining 23 for

the third knight to sit, so the total probability for this case is
Case 3: The second knight sits 3 or more spaces from the first knight. There are 20 possible

places for this out of 24, so the probability is . Then there are four places to put the last

knight out of 23, so the total probability for this case is

So add the probabilities to get the total:

Problem 8

What is the largest 2-digit prime factor of the integer ?

Solution

Expanding the binomial coefficient, we get . Let the prime be ; then


. If , then the factor of appears twice in the denominator. Thus, we
need to appear as a factor three times in the numerator, or . The largest such
prime is , which is our answer.

Problem 9

Find the minimum value of for .

Solution
Solution 1

Let . We can rewrite the expression as .

Since and because , we have . So we can apply AM-GM:


The equality holds when .

Therefore, the minimum value is (when ; since is continuous and

increasing on the interval and its range on that interval is from ,


by the Intermediate Value Theorem this value is attainable).

Solution 2

We can rewrite the numerator to be a perfect square by adding . Thus, we must


also add back .

This results in .

Thus, if , then the minimum is obviously . We show this possible with


the same methods in Solution 1; thus the answer is .

Solution 3

Let and rewrite the expression as , similar to the previous


solution. To minimize , take the derivative of and set it equal to zero.

The derivative of , using the Power Rule, is

is zero only when or . It can further be verified that and are


relative minima by finding the derivatives of other points near the critical points. However,

since is always positive in the given domain, . Therefore, = , and the

answer is .
Problem 10

The numbers , , and have something in common. Each is a four-digit number


beginning with that has exactly two identical digits. How many such numbers are there?

Solution

Suppose the two identical digits are both one. Since the thousands digits must be one, the
other one can be in only one of three digits,

Because the number must have exactly two identical digits, , , and .
Hence, there are numbers of this form.

Suppose the two identical digits are not one. Therefore, consider the following possibilities,

Again, , , and . There are numbers of this form as well.

Thus, the desired answer is .

Problem 11

The solid shown has a square base of side length . The upper edge is parallel to the base and
has length . All other edges have length . Given that , what is the volume of the
solid?
Solution
Solution 1

First, we find the height of the figure by drawing a perpendicular from the midpoint of
to . The hypotenuse of the triangle is the median of equilateral triangle , and one of
the legs is . We apply the Pythagorean Theorem to find that the height is equal to .

Next, we complete the figure into a triangular prism, and find the volume, which is

Now, we subtract off the two extra pyramids that we included, whose combined volume is

Thus, our answer is .

Solution 2
Extend and to meet at , and and to meet at . Now, we have a regular
tetrahedron , which has twice the volume of our original solid. This tetrahedron has
side length . Using the formula for the volume of a regular tetrahedron, which is

, where S is the side length of the tetrahedron, the volume of our original solid
is:

Problem 12

The length of diameter is a two digit integer. Reversing the digits gives the length of a
perpendicular chord . The distance from their intersection point to the center is a
positive rational number. Determine the length of .

Solution

Let and . It follows that and

. Applying the Pythagorean Theorem on and ,

Because is a positive rational number, the quantity cannot contain


any square roots. Either or must be 11. However, cannot be 11, because
both must be digits. Therefore, must equal eleven and must be a perfect square
(since ). The only pair that satisfies this condition is , so our
answer is .
Problem 13

For and each of its non-empty subsets, an alternating sum is defined as


follows. Arrange the number in the subset in decreasing order and then, beginning with the
largest, alternately add and subtract succesive numbers. For example, the alternating sum for
is and for it is simply . Find the sum of all such
alternating sums for .

Solution 1

Let be a non- empty subset of .

Then the alternating sum of plus the alternating sum of with 7 included is 7. In
mathematical terms, . This is true because when we take an alternating sum,
each term of has the opposite sign of each corresponding term of .

Because there are of these pairs, the sum of all possible subsets of our given set is .
However, we forgot to include the subset that only contains , so our answer is
.

Solution 2

Consider a given subset of that contains 7; then there is a subset which contains all
the elements of except for 7, and only those. Since each element of has one element
fewer preceding it than it does in , their signs are opposite; so the sum of the alternating
sums of and is equal to 7. There are subsets containing 7, so our answer is
.

Problem 14

In the adjoining figure, two circles with radii and are drawn with their centers units
apart. A to , one of the points of intersection, a line is drawn in such a way that the chords
and have equal length. ( is the midpoint of ) Find the area of the square with
a side length of .
Solution
Solution 1

First, notice that if we reflect over we get . Since we know that is on circle and
is on circle , we can reflect circle over to get another circle (centered at a new
point with radius ) that intersects circle at . The rest is just finding lengths:

Since is the midpoint of segment , is a median of triangle . Because we


know that , , and , we can find the third side of the
triangle using Stewart's Theorem or similar approaches. We get . So now we
have a kite with , , and , and all we
need is the length of the other diagonal . The easiest way it can be found is with the
Pythagorean Theorem. Let be the length of . Then

Doing routine algebra on the above equation, we find that , so

Solution 2

Draw additional lines as indicated. Note that since triangles and are isosceles,
the altitudes are also bisectors, so let .

Since triangles and are similar. If we let , we have


.

Applying the Pythagorean Theorem on triangle , we have . Similarly,


for triangle , we have .

Subtracting, .
Solution 3
Let . Angles , , and must add up to . By the Law of
Cosines, . Also, angles and equal and
. So we have

Taking the of both sides and simplifying using the cosine addition identity gives .

Solution 4

Observe that the length of the area where the two circles intersect can be found explicitly as
. Let , then the power of point with regards to the larger circle gives

Problem 15

The adjoining figure shows two intersecting chords in a circle, with on minor arc .
Suppose that the radius of the circle is , that , and that is bisected by .
Suppose further that is the only chord starting at which is bisected by . It follows
that the sine of the minor arc is a rational number. If this fraction is expressed as a

fraction in lowest terms, what is the product ?

Solution

Let be any fixed point on circle and let be a chord of circle . The locus of
midpoints of the chord is a circle , with diameter . Generally, the circle can
intersect the chord at two points, one point, or they may not have a point of intersection.
By the problem condition, however, the circle is tangent to BC at point N.

Let M be the midpoint of the chord . From right triangle ,

. Thus, .
Notice that the distance equals (Where
is the radius of circle P). Evaluating this,

From , we see that

Next, notice that . We can therefore apply the tangent


subtraction formula to obtain ,

. It follows that

, resulting in an answer of .

You might also like